任意の

とある問題の証明を読んでいたら,こんな一文に出会いました.記号の意味はさておき,\(\delta(A),\delta(\overline{A}),\epsilon\)はいずれも実数です.

(中略) \(\delta(\overline{A}) \leq \delta(A) + \epsilon\).\(\epsilon\)は任意の正数だから,\(\delta(\overline{A}) \leq \delta(A)\).

 

(ここで理解が詰まる)

「任意」と言われたから,どんな正数でもOKですが,でかい数をもってきても結論の不等式が得られるとは思えませんから,めちゃくちゃ小さい数を持ってくることにします.\(\delta(A)\)に加えられる数\(\epsilon\)をめちゃくちゃ小さくしたとしてもやっぱり\(\delta(\overline{A})\)よりも\(\delta(A)\)の方が大きい…,ということは\(\delta(A)\)の方が大きいと言える…のか…??感覚的には納得できるような気もしないでもないですが,でも小さいとはいえ正数を加えられている以上それを除いてもやはり\(\delta(\overline{A})\)以上だ,なんて言えるのだろうか,とも感じられ,釈然としません.

議論に関係のない文字がうるさいので,ちょっと簡略化して書き直します.

\(a,b \in \mathbb{R}\)とする.
任意の正数\(\epsilon\)に対して\(a \leq b + \epsilon\)が成り立つならば,\(a \leq b\)が成り立つ.

こう書くとちょっと高校数学の証明問題ぽいですね.実際,いちおう数学Aの集合と論理を終えた高1生なら理解できる(証明できる)と思います.調べてみましょう.

証明

背理法で示す.証明したいことは
\[\forall \epsilon >0 [a \leq b + \epsilon ]\Longrightarrow a \leq b\]
であるから,否定をとると
\begin{align*}
&\overline{\forall \epsilon >0 [a \leq b + \epsilon ]\Longrightarrow a \leq b}\\
\Longleftrightarrow~&\overline{\overline{\forall \epsilon >0 [a \leq b + \epsilon ]} \lor a \leq b}\\
\Longleftrightarrow~&\forall \epsilon >0 [a \leq b + \epsilon ] \land a > b
\end{align*}\(a>b\)だから,\(a-b > 0\).また,\(\forall \epsilon >0 [a \leq b + \epsilon ]\),つまり\(a \leq b + \epsilon\)が任意の\(\epsilon > 0\)について成り立つから,ここでは\(\epsilon=\frac{a-b}{2} >0\)ととることにする.すると,\[a \leq b +\frac{a-b}{2} \Longleftrightarrow a \leq b\]を得る.これは\(a>b\)であることに反する.

証明終

この証明,知識としてはほぼ高校数学の知識しか使ってない上にとてもシンプルな論証なので,教科書では無視しがちな「任意の」を重要性を確認させる問題としていいんじゃないかな,なんて思いました.「任意の」と言っているのだから,都合のよい\(\epsilon\)を代入したところがポイントです.

◆値域の問題(別解)

\(4x^2-8xy+10y^2=1\)のとき,\(x^2+y^2\)の最大値と最小値を求めよ.

\(x^2+y^2\)がとりうる値の範囲を\(\mathcal{R}\)とおく.
\begin{align*}
&k \in \mathcal{R}\\
\Longleftrightarrow~&\exists x \exists y \begin{cases} x^2+y^2=k \\ 4x^2-8xy+10y^2=1\end{cases}\\
\Longleftrightarrow~&\exists x \exists y \left[\begin{cases} x^2+y^2=k \\ 4x^2-8xy+10y^2=1\end{cases} \land \exists r \exists \theta \begin{cases} x=r\cos\theta \\ y= r\sin \theta \end{cases}\right]\\
\Longleftrightarrow~&\exists x \exists y \exists r \exists \theta \left[\begin{cases} x^2+y^2=k \\ 4x^2-8xy+10y^2=1\end{cases} \land \begin{cases} x=r\cos\theta \\ y= r\sin \theta \end{cases}\right]\\
\Longleftrightarrow~&\exists x \exists y \exists r \exists \theta \left[\begin{cases} r^2=k \\ 4r^2\cos^2\theta-8r^2\cos\theta\sin\theta +10r^2\sin^2\theta=1\end{cases} \land \begin{cases} x=r\cos\theta \\ y= r\sin \theta \end{cases}\right]\\
\Longleftrightarrow~&\exists x \exists y \exists r \exists \theta \left[\begin{cases} r^2=k \\ 4k\cos^2\theta-8k\cos\theta\sin\theta +10k\sin^2\theta=1\end{cases} \land \begin{cases} x=r\cos\theta \\ y= r\sin \theta \end{cases}\right]\\
\Longleftrightarrow~&\exists r \exists \theta \left[\begin{cases} r^2=k \\ 4k\cos^2\theta-8k\cos\theta\sin\theta +10k\sin^2\theta=1\end{cases} \land \exists x \exists y \begin{cases} x=r\cos\theta \\ y= r\sin \theta \end{cases}\right]\\
\Longleftrightarrow~&\exists r \exists \theta \begin{cases} r^2=k \\ 4k\cos^2\theta-8k\cos\theta\sin\theta +10k\sin^2\theta=1\end{cases}\\
\Longleftrightarrow~&\exists r \begin{cases} r^2=k \\ \exists \theta [4k\cos^2\theta-8k\cos\theta\sin\theta +10k\sin^2\theta=1]\end{cases}\\
\Longleftrightarrow~&\exists r \begin{cases} r^2=k \\ \exists \theta [7k-4k\sin 2\theta-3k\cos 2\theta=1]\end{cases}\\
\Longleftrightarrow~&\exists r \begin{cases} r^2=k \\ \exists \theta [3k\cos 2\theta + 4k\sin 2\theta=7k-1]\end{cases}\\
\Longleftrightarrow~&\exists r \begin{cases} r^2=k \\ \exists \theta \left[\left(\begin{array}{c}3k \\ 4k \\ \end{array}\right) \cdot \left(\begin{array}{c}\cos 2\theta \\ \sin 2\theta \\ \end{array}\right)=7k-1 \right]\end{cases}\\
\Longleftrightarrow~&\exists r \begin{cases} r^2=k \\ \exists \alpha \left[5k\cos \alpha =7k-1 \right]\end{cases}\qquad\text{(\(\alpha\)は\(\left(\begin{array}{c}3k \\ 4k \\ \end{array}\right)\)と\(\left(\begin{array}{c}\cos 2\theta \\ \sin 2\theta \\ \end{array}\right)\)のなす角)}\\
\Longleftrightarrow~&\exists r \begin{cases} r^2=k \\ -1 \leq \frac{7k-1}{5k} \leq 1\end{cases}\\
\Longleftrightarrow~&\exists r [r^2=k] \land -1 \leq \frac{7k-1}{5k} \leq 1\\
\Longleftrightarrow~& k \geq 0 \land -1 \leq \frac{7k-1}{5k} \leq 1\\
\Longleftrightarrow~& k > 0 \land -5k \leq 7k-1 \leq 5k\\
\Longleftrightarrow~& k > 0 \land \frac{1}{12} \leq k \leq \frac{1}{2}\\
\Longleftrightarrow~& \frac{1}{12} \leq k \leq \frac{1}{2}\\
\end{align*}

ゆえに,最大値\(\frac{1}{2}\),最小値\(\frac{1}{12}\).

\(\ast\)    \(\ast\)    \(\ast\)

はじめに\(x=r\cos \theta,y=r\sin \theta\)とおき,そして「積の和(1次結合)」を「内積」と見なして処理してみました(合成でもいいと思いますが).前回の解法と違い,どの行も同値変形なので逆の考察は必要ありません.たぶんこっちの解法のほうが簡単だと思いますがどうでしょう.

◆値域の問題(つづき)

\begin{align*}
\Longrightarrow~&k=\frac{1}{4} \lor \exists x \exists y \begin{cases} x^2+y^2=k \\ k \neq 0 \land y\neq 0 \\ \exists t \left[ (4k-1)t^2-8kt+(10k-1)=0 \right] \land \exists t \left [\frac{x}{y}=t \right]\end{cases}\\
\Longleftrightarrow~&k=\frac{1}{4} \lor \exists x \exists y \begin{cases} x^2+y^2=k \\ k \neq 0 \land y\neq 0 \\ \exists t \left[ (4k-1)t^2-8kt+(10k-1)=0 \right]\end{cases}\\
\Longleftrightarrow~&k=\frac{1}{4} \lor \exists x \exists y \begin{cases} x^2+y^2=k \\ k \neq 0 \land y\neq 0 \\ \exists t \left[ (4k-1)t^2-8kt+(10k-1)=0 \land (k=\frac{1}{4} \lor k\neq \frac{1}{4})\right]\end{cases}\\
\Longleftrightarrow~&k=\frac{1}{4} \lor \exists x \exists y \begin{cases} x^2+y^2=k \\ k \neq 0 \land y\neq 0 \\ \exists t \left[ (4k-1)t^2-8kt+(10k-1)=0 \land k=\frac{1}{4}\right] \\
\lor \exists t \left[(4k-1)t^2-8kt+(10k-1)=0 \land k\neq \frac{1}{4}\right]\end{cases}\\
\Longleftrightarrow~&k=\frac{1}{4} \lor \exists x \exists y \begin{cases} x^2+y^2=k \\ k \neq 0 \land y\neq 0 \\ \exists t \left[ t=\frac{3}{4} \land k=\frac{1}{4}\right] \lor \exists t \left[\frac{1}{12} \leq k \leq \frac{1}{2} \land k\neq \frac{1}{4}\right]\end{cases}\\
\Longleftrightarrow~&k=\frac{1}{4} \lor \exists x \exists y \begin{cases} x^2+y^2=k \\ k \neq 0 \land y\neq 0 \\ k=\frac{1}{4} \lor \left( \frac{1}{12} \leq k \leq \frac{1}{2} \land k\neq \frac{1}{4}\right)\end{cases}\\
\Longleftrightarrow~&k=\frac{1}{4} \lor \begin{cases} \exists x \exists y ( x^2+y^2=k \land y\neq 0 ) \\ k \neq 0 \\ \frac{1}{12} \leq k \leq \frac{1}{2} \end{cases}\\
\Longleftrightarrow~&k=\frac{1}{4} \lor \begin{cases} \exists x \exists y ( x^2+y^2=k \land y\neq 0 ) \\ \frac{1}{12} \leq k \leq \frac{1}{2} \end{cases}\\
\Longleftrightarrow~&k=\frac{1}{4} \lor \left( k \geq 0 \land \frac{1}{12} \leq k \leq \frac{1}{2} \right)\\
\Longleftrightarrow~&\left( k=\frac{1}{4} \lor k \geq 0 \right) \land \left( k=\frac{1}{4} \lor \frac{1}{12} \leq k \leq \frac{1}{2} \right)\\
\Longleftrightarrow~&k \geq 0 \land \frac{1}{12} \leq k \leq \frac{1}{2}\\
\Longleftrightarrow~&\frac{1}{12} \leq k \leq \frac{1}{2}
\end{align*}

逆に,\(k=\frac{1}{12}\)のとき,前記事\((\ast)\)が成り立つかを調べる.
\begin{align*}
&\exists x \exists y \begin{cases} x^2+y^2=k \\ k \neq 0 \land y\neq 0 \\ \exists t \left[ (4k-1)t^2-8kt+(10k-1)=0\land \frac{x}{y}=t \right]\end{cases}\tag{\(\ast\)}\\
\Longleftrightarrow~&\exists x \exists y \begin{cases} x^2+y^2=\frac{1}{12} \\ \frac{1}{12} \neq 0 \land y\neq 0 \\ \exists t \left[ 4t^2+4t+1=0\land \frac{x}{y}=t \right]\end{cases}\\
\Longleftrightarrow~&\exists x \exists y \begin{cases} x^2+y^2=\frac{1}{12} \\ y\neq 0 \\ \exists t \left[ t=-\frac{1}{2}\right] \land y=-2x \end{cases}\\
\Longleftrightarrow~&\exists x \exists y \begin{cases} x^2+y^2=\frac{1}{12} \\ y=-2x \\ y\neq 0 \end{cases}
\end{align*}
この命題は明らかに真である.

\(k=\frac{1}{2}\)のときも同様に\((\ast)\)は真となる.したがって最大値は\(\frac{1}{2}\),最小値は\(\frac{1}{12}\).

◆値域の問題

\(4x^2-8xy+10y^2=1\)のとき,\(x^2+y^2\)の最大値と最小値を求めよ.

\(x^2+y^2\)がとりうる値の範囲を\(\mathcal{R}\)とおく.
\[
\begin{align*}
&k \in \mathcal{R}\\
\Longleftrightarrow~&\exists x \exists y \begin{cases} x^2+y^2=k \\ 4x^2-8xy+10y^2=1\end{cases}\\
\Longleftrightarrow~&\exists x \exists y \begin{cases} x^2+y^2=k \\ 4x^2-8xy+10y^2=1\end{cases} \land (k=0 \lor k \neq 0)\\
\Longleftrightarrow~&\exists x \exists y \left[\begin{cases} x^2+y^2=k \\ 4x^2-8xy+10y^2=1 \\ k=0 \end{cases} \lor \begin{cases} x^2+y^2=k \\ 4x^2-8xy+10y^2=1 \\ k\neq 0 \end{cases}\right]\\
\Longleftrightarrow~&\exists x \exists y \left[\begin{cases} x=y=0 \\ 4x^2-8xy+10y^2=1 \\ k=0 \end{cases} \lor \begin{cases} x^2+y^2=k \\ 4kx^2-8kxy+10ky^2=k \\ k\neq 0 \end{cases}\right]\\
\Longleftrightarrow~&\exists x \exists y \begin{cases} x^2+y^2=k \\ (4k-1)x^2-8kxy+(10k-1)y^2=0 \\ k \neq 0\end{cases}\\
\Longleftrightarrow~&\exists x \exists y \begin{cases} x^2+y^2=k \\ (4k-1)x^2-8kxy+(10k-1)y^2=0 \\ k \neq 0\end{cases} \land (y=0 \lor y \neq 0)\\
\Longleftrightarrow~&\exists x \exists y \left[\begin{cases} x^2+y^2=k \\ (4k-1)x^2-8kxy+(10k-1)y^2=0 \\ k \neq 0 \\ y=0 \end{cases} \right.\\
&\left.\lor \begin{cases} x^2+y^2=k \\ (4k-1)x^2-8kxy+(10k-1)y^2=0 \\ k \neq 0 \\ y\neq 0 \end{cases}\right]\\
\Longleftrightarrow~&\exists x \exists y \left[\begin{cases} x^2=k \\ (4k-1)x^2=0 \\ k \neq 0\\ y=0 \end{cases} \lor \begin{cases} x^2+y^2=k \\ (4k-1)x^2-8kxy+(10k-1)y^2=0 \\ k \neq 0 \\ y\neq 0 \end{cases}\right]\\
\Longleftrightarrow~&\exists x \exists y \left[\begin{cases} x^2=k \\ (4k-1)k=0 \\ k \neq 0\\ y=0 \end{cases} \lor \begin{cases} x^2+y^2=k \\ (4k-1)\left(\frac{x}{y}\right)^2-8k\frac{x}{y}+(10k-1)=0 \\ k \neq 0 \\ y\neq 0 \end{cases}\right]\\
\Longleftrightarrow~&\exists x \exists y \left[\begin{cases} x^2=\frac{1}{4} \\ k=\frac{1}{4} \\ k \neq 0\\ y=0 \end{cases} \lor \begin{cases} x^2+y^2=k \\ (4k-1)\left(\frac{x}{y}\right)^2-8k\frac{x}{y}+(10k-1)=0 \\ k \neq 0 \\ y\neq 0 \end{cases}\right]\\
\Longleftrightarrow~&k=\frac{1}{4} \lor \exists x \exists y \begin{cases} x^2=\frac{1}{4} \\ y=0 \end{cases} \lor \exists x \exists y\begin{cases} x^2+y^2=k \\ (4k-1)\left(\frac{x}{y}\right)^2-8k\frac{x}{y}+(10k-1)=0 \\ k \neq 0 \land y\neq 0 \end{cases}\\
\Longleftrightarrow~&k=\frac{1}{4} \lor \exists x \exists y\begin{cases} x^2+y^2=k \\ (4k-1)\left(\frac{x}{y}\right)^2-8k\frac{x}{y}+(10k-1)=0 \\ k \neq 0 \land y\neq 0 \end{cases}\\
\Longleftrightarrow~&k=\frac{1}{4} \lor \exists x \exists y \left[\begin{cases} x^2+y^2=k \\ (4k-1)\left(\frac{x}{y}\right)^2-8k\frac{x}{y}+(10k-1)=0 \\ k \neq 0 \land y\neq 0 \end{cases} \land \exists t \left[\frac{x}{y}=t\right]\right]\\
\Longleftrightarrow~&k=\frac{1}{4} \lor \exists x \exists y \exists t \begin{cases} x^2+y^2=k \\ (4k-1)\left(\frac{x}{y}\right)^2-8k\frac{x}{y}+(10k-1)=0 \\ k \neq 0 \land y\neq 0 \\ \frac{x}{y}=t\end{cases}\\
\Longleftrightarrow~&k=\frac{1}{4} \lor \exists x \exists y \exists t \begin{cases} x^2+y^2=k \\ (4k-1)t^2-8kt+(10k-1)=0 \\ k \neq 0 \land y\neq 0 \\ \frac{x}{y}=t\end{cases}\\
\Longleftrightarrow~&k=\frac{1}{4} \lor \exists x \exists y \begin{cases} x^2+y^2=k \\ k \neq 0 \land y\neq 0 \\ \exists t \left[ (4k-1)t^2-8kt+(10k-1)=0\land \frac{x}{y}=t \right]\end{cases}\tag{\(\ast\)}\
\end{align*}
\]
ここで,\(\exists x [p(x) \land q(x)] \Longrightarrow \exists x p(x) \land \exists x q(x)\)であることに注意して,(つづき

◆円と放物線(別解その2)

\(y=x^2+k\)(\(k\)は定数)と円\(x^2+y^2=4\)について,
\((1)\)異なる\(4\)つの共有点をもつとき,定数\(k\)の値の範囲を求めよ.
\((2)\)放物線と円が接するとき,定数\(k\)の値を求めよ.

円と放物線が共有点をもつときの\(k\)の範囲を\(\mathcal{D}\)とおく.
\begin{align*}
&k\in\mathcal{D}\\
\Longleftrightarrow~ &\exists x \exists y \begin{cases}x^2+y^2=4 \\ y=x^2+k\end{cases}\\
\Longleftrightarrow~ &\exists x \exists y \begin{cases}x^2+y^2=4 \\ y=(4-y^2)+k\end{cases}\\
\Longleftrightarrow~ &\exists x \exists y \begin{cases}x^2=4-y^2 \\ y^2+y-4-k=0\end{cases}\\
\Longleftrightarrow~ &\exists y\left[\exists x [x^2=4-y^2] \land y^2+y-4-k=0 \right]\\
\Longleftrightarrow~ &\exists y\left[-2 \leq y \leq 2 \land y^2+y-4=k \right]\\
\Longleftrightarrow~ &\exists y\left[-2 \leq y \leq 2 \land \left(y+\frac{1}{2}\right)^2-\frac{17}{4}=k \right]\\
\Longleftrightarrow~ &-\frac{17}{4} \leq k \leq 2 \tag{\(\ast\)}
\end{align*}
\((2)~\)上の結果と下図から,接するとき,\(k=\pm 2\)または\(k=-\frac{17}{4}\).


\((1)~\)\((2)\)の考察と上図から,異なる\(4\)つの共有点をもつとき,\(-\frac{17}{4} < k < -2\).

\((\ast)\)の考察は下図による(文字定数は分離せよ,の方針).

文字は死んで変域残す

\(x,y\)が実数で,\(2x^2+3xy+2y^2=1\)を満たすとき,\(x+y+xy\)の最大値と最小値を求めよ.

という定番の問題についてみてみます.これは\(x+y=u,xy=v\)とおいたあと,「\(x,y\)が実数」という条件を\(u,v\)に反映させるのがポイントなのでした.すなわち,\(t^2-ut+v=0\)の判別式を\(\geq 0\)とすることにより
\[u^2-4v \geq 0\]
この不等式に注意しながら\(x+y+xy\)の最大値・最小値を調べる,という流れが定石でした(虎は死んで皮を残す,人は死んで名を残す,文字は死んで変域を残す…).この,\(u^2-4v \geq 0\)を得る流れは論理的にはどうなっているのか,調べてみます.

解答
\begin{align*}
&\text{\(x+y+xy\)が\(k\)という値をとる}\\
\Longleftrightarrow~&\exists x \exists y[x+y+xy=k \land 2x^2+3xy+2y^2=1]\\
\Longleftrightarrow~&\exists x \exists y \left[x+y+xy=k \land 2x^2+3xy+2y^2=1 \land \exists u \exists v \begin{cases}x+y=u\\xy=v\end{cases}\right]\\
\Longleftrightarrow~&\exists x \exists y \exists u \exists v\left[x+y+xy=k \land 2(x+y)^2-xy=1 \land \begin{cases}x+y=u\\xy=v\end{cases}\right]\\
\Longleftrightarrow~&\exists x \exists y \exists u \exists v\left[u+v=k \land 2u^2-v=1 \land \begin{cases}x+y=u\\xy=v\end{cases}\right]\\
\Longleftrightarrow~&\exists u \exists v\left[u+v=k \land 2u^2-v=1 \land \exists x \exists y \begin{cases}x+y=u\\xy=v\end{cases}\right]\tag{\(\ast\)}
\end{align*}
ここで
\begin{align*}
&\exists x \exists y \begin{cases}x+y=u\\xy=v\end{cases}\\
\Longleftrightarrow~&\exists x \exists y \begin{cases}x+y=u\\ \frac{1}{4}((x+y)^2-(x-y)^2)=v\end{cases}\\
\Longleftrightarrow~&\exists x \exists y \begin{cases}x+y=u \\ (x-y)^2=u^2-4v\end{cases}\\
\Longleftrightarrow~&\exists x \exists y \begin{cases}x+y=u \\ |x-y|=\sqrt{u^2-4v}\end{cases}\\
\Longleftrightarrow~&\exists x \exists y \begin{cases}y=u-x \\ (x-y=\sqrt{u^2-4v} \land x-y \geq 0 ) \lor (y-x=\sqrt{u^2-4v} \land x-y < 0 )\end{cases}\\ \Longleftrightarrow~&\exists x \left[\left(x=\frac{1}{2}\left(u+\sqrt{u^2-4v}\right) \land x \geq \frac{u}{2} \right) \lor \left(x=\frac{1}{2}\left(u-\sqrt{u^2-4v}\right) \land x < \frac{u}{2} \right)\right]\\ \Longleftrightarrow~&\frac{1}{2}\left(u+\sqrt{u^2-4v}\right) \geq \frac{u}{2} \lor \frac{1}{2}\left(u-\sqrt{u^2-4v} \right) < \frac{u}{2} \\ \Longleftrightarrow~&\sqrt{u^2-4v} \geq 0 \lor \sqrt{u^2-4v} < 0\\ \Longleftrightarrow~&\sqrt{u^2-4v} \geq 0\\ \Longleftrightarrow~&u^2-4v \geq 0\\ \end{align*} であるから
\begin{align*}
(\ast)\Longleftrightarrow~&\exists u \exists v\left[u+v=k \land v=2u^2-1 \land u^2-4v \geq 0\right]\\
\Longleftrightarrow~&\exists u \left[u+(2u^2-1)=k \land u^2-4(2u^2-1) \geq 0\right]\\
\Longleftrightarrow~&\exists u \left[k = 2u^2+u-1 \land -\frac{2}{\sqrt{7}} \leq u \leq \frac{2}{\sqrt{7}}\right]\\
\Longleftrightarrow~&-\frac{9}{8} \leq u \leq \frac{1}{7}+\frac{2}{\sqrt{7}}
\end{align*}
ゆえに,最大値\(\displaystyle \frac{1}{7}+\frac{2}{\sqrt{7}}\),最小値\(\displaystyle -\frac{9}{8}\).

\(\ast\)    \(\ast\)    \(\ast\)

ここで,~であるから」までがいわゆる「実数の存在条件」の処理です.前回の「\(m^2<\frac{1}{12}\)を満たす実数の存在条件は~」と同じ考え方で導出してみました.

◆無理不等式その2

次の式を\(\sqrt{\quad}\)のない形で表せ(同値変形せよ).
\[\sqrt{a} < b\]

恒真条件の追加と分配法則,矛盾命題の消去により,
\begin{align*}
&\sqrt{a} < b\\ \Longleftrightarrow~&\sqrt{a}< b \land (b \geq 0 \lor b < 0)\\ \Longleftrightarrow~&(\sqrt{a} < b \land b \geq 0)\lor (\sqrt{a} < b \land b < 0)\\ \Longleftrightarrow~&\sqrt{a} < b \land b \geq 0 \end{align*} ここからさらに変形を考えますが,前回同様,いきなり同値な変形は考えづらいので,必要性\((\Rightarrow)\)と十分性\((\Leftarrow)\)を別々に考えることにします. まず必要性\((\Rightarrow)\)から.\(\sqrt{a} \geq 0\)ですから,\(\sqrt{a} < b\)の両辺を2乗することができて,例えば次のように必要条件が得られます: \begin{align*} &\sqrt{a} < b \land b \geq 0 \Longrightarrow a < b^2 \land b \geq 0 \tag{1} \end{align*} 次にこの\((1)\)における十分性\((\Leftarrow)\)を考えてみます.当然,\(a < b^2\)の両辺に\(\sqrt{\quad}\)をとりたくなりますが,しかし\(a\)が正である保証は今手元の仮定にはありません.つまり\(\sqrt{\quad}\)をとることができず,戻れない.そこで,\((1)\)において必要条件をもう少し絞り出すことを考えます.欲しいのは\(a \geq 0\)ですが,\(\sqrt{a}\)の‘中身’は正ですから,必要条件は \[\sqrt{a} < b\land b \geq 0 \Longrightarrow a < b^2 \land b \geq 0 \land a \geq 0\] とできるはずです.そして改めて十分性を確認してみます. \begin{align*} a < b^2 \land b \geq 0 \land a \geq 0 \Longrightarrow &\sqrt{a} < \sqrt{b^2} \land b\geq 0 \land a \geq 0\\ \Longrightarrow &\sqrt{a} < |b| \land b \geq 0 \land a \geq 0\\ \Longrightarrow &\sqrt{a} < b \land b \geq 0 \land a \geq 0\\ \Longrightarrow &\sqrt{a} < b \land b \geq 0 \end{align*} となり戻れました.これで必要十分(同値)であることが分かりました.したがって\((1)\)の論理式は, \[\sqrt{a} < b \land b \geq 0 \Longleftrightarrow a < b^2 \land b \geq 0 \land a \geq 0 \Longleftrightarrow 0\leq a < b^2 \land b \geq 0 \] と書きかえれば同値になることが分かりました. 以上により,

\[\sqrt{a} < b \Longleftrightarrow 0\leq a < b^2 \land b \geq 0 \]

と同値変形できることが分かりました.

◆無理不等式その1

次の式を\(\sqrt{\quad}\)のない形で表せ(同値変形せよ).
\[\sqrt{a}>b\]

恒真条件の追加と分配法則により,
\begin{align*}
&\sqrt{a}>b\\
\Longleftrightarrow~&\sqrt{a}>b \land (b \geq 0 \lor b < 0)\\
\Longleftrightarrow~& (\sqrt{a}>b \land b \geq 0)\text{(ア)} \lor (\sqrt{a}>b \land b < 0) \text{(イ)}
\end{align*}

(ア)と(イ)を分けて考えます.

(まず(ア)について)
いきなり同値な変形は考えづらいので,必要性\((\Rightarrow)\)と十分性\((\Leftarrow)\)を別々に考えることにします. まず必要性\((\Rightarrow)\)から.今,\(b \geq 0\)ですから,\(\sqrt{a}>b\)の両辺を2乗することができて,例えば
\begin{align*}
&\sqrt{a}>b \land b \geq 0 \text{(ア)}\Longrightarrow a > b^2\tag{1}
\end{align*}
のように必要条件が得られます.次にこの\((1)\)における十分性\((\Leftarrow)\)を考えてみましょう.両辺が正ですから,\(\sqrt{\quad}\)をとることができますが,
\[a > b^2 \Longrightarrow \sqrt{a} > \sqrt{b^2} \Longrightarrow \sqrt{a} > |b|\]
となり(ア)に戻れません(\(b\)の正負がわからない).そこで,\((1)\)において(ア)の必要条件をもう少し絞り出しておきましょう.
\[\sqrt{a} > b\land b \geq 0 \text{(ア)}\Longrightarrow a > b^2 \land b \geq 0\]
そして十分性を確認してみます.
\begin{align*}
a > b^2 \land b \geq 0\Longrightarrow &\sqrt{a} > \sqrt{b^2} \land b\geq 0 \\
\Longrightarrow &\sqrt{a} > |b| \land b \geq 0 \\
\Longrightarrow &\sqrt{a} > b \land b \geq 0
\end{align*}
となりこれなら(ア)に戻れます.これで必要十分(同値)であることが分かりました.したがって\((1)\)の論理式は,
\[\sqrt{a} > b \land b \geq 0 \text{(ア)}\Longleftrightarrow a > b^2 \land b \geq 0 \tag{1′}\]
と書きかえれば同値になることが分かりました.

(次に(イ)について)
必要性\(\Rightarrow\)から見てみます.ここでは例えば明らかな必要性
\[(\sqrt{a} > b \land b < 0) \text{(イ)} \Longrightarrow b < 0 \tag{2}\] を考えてみます.逆(十分性)はどうか? \[b < 0 \Longrightarrow (\sqrt{a} > b \land b < 0) \text{(イ)}\]が言えるか?…残念ながら言えません.なぜなら\(a\)は\(\sqrt{\quad}\)の中にあるのだから正でなくてはなりませんが,しかし仮定には\(a\)の正負についての言及がないからです.このことを踏まえて\((2)\)で(イ)の必要条件を適切に絞り出しておきます.\(\sqrt{\quad}\)の‘中身’は正であることに着目して, \[(\sqrt{a} > b \land b < 0) \text{(イ)} \Longrightarrow a \geq 0 \land b < 0\] さてこれならどうでしょうか?逆(十分性)を見てみると \[a \geq 0 \land b < 0 \Longrightarrow (\sqrt{a} > b \land b < 0) \text{(イ)}\] は確かに言えます.したがって,\((2)\)の論理式は \[(\sqrt{a} > b \land b < 0) \text{(イ)} \Longleftrightarrow a \geq 0 \land b < 0\tag{2'}\] と書けば同値であることがわかりました. \((1'),(2')\)により,

\[\sqrt{a}>b \Longleftrightarrow (a > b^2 \land b \geq 0) \lor (a \geq 0 \land b < 0)\]

と同値変形できることが分かりました.

ちなみにもし,\(b \geq 0\)という条件を‘大前提’として奉れば,当然
\[\sqrt{a}>b \Longleftrightarrow a > b^2 \land b \geq 0\]
と書けます.

◆軌跡と同値変形その2

とある軌跡の問題の模範解答に関して次のような質問がありました.

模範解答では,\(X=\frac{16m^2}{4m^2+1}\)を得たのち
\[\text{「\(m^2<\frac{1}{12}\)を満たす実数の存在条件は\(0 \leq m^2<\frac{1}{12}\)だから,\(0 \leq X <1\)」}\]
とあるが,これは論理的にはどのように導いたのか?

 

模範解答だとたしかに何をやっているのか,というか何をいっているのかよくわかりません.

そこで論理式で考えます.模範解答の論理の流れはおそらくは以下です(代入法による解答との分岐点は\((1)\)からです):

\begin{align*}
&\exists m \left[X=\frac{16m^2}{4m^2+1} \land Y=m(X-4)\land m^2<\frac{1}{12}\right]\\
\Longleftrightarrow~&\exists m\left[X=\frac{16m^2}{4m^2+1} \land Y=m(X-4)\land m^2<\frac{1}{12} \land (X=4 \lor X \neq 4)\right]\\
\Longleftrightarrow~&\exists m\left[\left(X=\frac{16m^2}{4m^2+1} \land Y=m(X-4)\land 0\leq m^2<\frac{1}{12} \land X=4 \right)\right.\\
&\lor \left.\left(X=\frac{16m^2}{4m^2+1} \land Y=m(X-4)\land m^2<\frac{1}{12} \land X \neq 4\right)\right]\\
\Longleftrightarrow~&\exists m\left[X=\frac{16m^2}{4m^2+1} \land m=\frac{Y}{X-4}\land m^2<\frac{1}{12} \land X \neq 4\right]\tag{0}\\
\Longleftrightarrow~&\exists m\left[X=\frac{16m^2}{4m^2+1} \land m=\frac{Y}{X-4}\land m^2<\frac{1}{12} \land X \neq 4 \land \exists t\geq 0 [t=m^2]\right]\tag{1}\\
\Longleftrightarrow~&\exists m\left[X=\frac{16m^2}{4m^2+1} \land m=\frac{Y}{X-4}\land m^2<\frac{1}{12} \land X \neq 4 \land \exists t[t\geq 0 \land t=m^2]\right]\tag{2}\\
\Longleftrightarrow~&\exists m \exists t\left[X=\frac{16m^2}{4m^2+1} \land m=\frac{Y}{X-4}\land m^2<\frac{1}{12} \land X \neq 4 \land t \geq 0 \land t=m^2\right]\tag{3}\\
\Longleftrightarrow~&\exists m \exists t\left[X=\frac{16t}{4t+1} \land m=\frac{Y}{X-4}\land t<\frac{1}{12} \land X \neq 4 \land t \geq 0 \land t=m^2\right]\tag{4}\\
\Longleftrightarrow~&\exists t\left[t=\frac{1}{4}\left(\frac{X}{4-X}\right) \land 0\leq t<\frac{1}{12} \land X \neq 4 \land t=\left(\frac{Y}{X-4}\right)^2\right]\tag{5}\\
\Longleftrightarrow~&0\leq \frac{1}{4}\left(\frac{X}{4-X}\right)<\frac{1}{12} \land X \neq 4 \land \frac{1}{4}\left(\frac{X}{4-X}\right)=\left(\frac{Y}{X-4}\right)^2\tag{6}\\
\Longleftrightarrow~&\frac{(X-2)^2}{4}+Y^2 = 1 \land 0 \leq X < 1 \land X \neq 4 \\
\Longleftrightarrow~&\frac{(X-2)^2}{4}+Y^2 = 1 \land 0 \leq X < 1
\end{align*}

\((0)\)ここまでこれと同じ
\((1)\)恒真条件\(\exists t \geq 0 [t=m^2]\)の追加.\(\exists m\)の支配域の下で考えているので,当然\(m^2\)すなわち\(0\)以上の\(t\)が存在しますから,恒真条件です.
\((2)\)略記の書き直し
\((3)\)\(\exists t\)の支配域の変更.自分\((t=m^2)\)以外はどれも変数\(t\)を含んでいないのでいちばん外側に追い出せます.
\((4)\)代入法(\(m^2\)に\(t\)を代入)
\((5)\)\(\exists m\)の処理
\((6)\)\(\exists t\)の処理

こうしてみると,模範解答での「(\(X=\frac{16m^2}{4m^2+1}\)において)\(m^2<\frac{1}{12}\)を満たす実数の存在条件は\(0 \leq m^2<\frac{1}{12}\)だから,\(0 \leq X <1\)」とは,厳密には以下のような操作を指して言っているのだと分かります:

    • \(m^2\)を\(t\)とおき\(\left((0) \Leftrightarrow (1)\right)\),
    • 支配域の変更を経て\(\left((3)\right)\),
    • 代入法により\(X=\frac{16t}{4t+1}\left(\Leftrightarrow t=\frac{1}{4}\left(\frac{X}{4-X}\right)\right)\)と(★)\(0\leq t<\frac{1}{12}\)が出現するわけですが\(\left((4)\right)\),
    • 前者は\(t\)についての1次式ですから,\(\exists t\)により\(t\)を同値変形として「消去」できて,\(\left((5) \Leftrightarrow (6)\right)\)
    • 結果,(★★)同値性を保ったまま\(0 \leq X < 1\)が得られる.

模範解答にある
\[\text{「\(m^2<\frac{1}{12}\)を満たす実数の存在条件は\(0 \leq m^2<\frac{1}{12}\)だから,」}\]
に対応する部分が(★),そして
\[\text{「\(0 \leq X <1\)」}\]
に対応する部分が(★★),であろうとおそらくは考えられます.…確かに,このような内容になると「端折る」ことも必要になってくるのかもしれません.

他方,次のような「置き換え」をしない変形も考えられます.

(別解釈)
\begin{align*}
&\exists m \left[X=\frac{16m^2}{4m^2+1} \land Y=m(X-4)\land m^2<\frac{1}{12}\right]\\
\Longleftrightarrow~&\exists m\left[X=\frac{16m^2}{4m^2+1} \land Y=m(X-4)\land m^2<\frac{1}{12} \land (X=4 \lor X \neq 4)\right]\\
\Longleftrightarrow~&\exists m\left[\left(X=\frac{16m^2}{4m^2+1} \land Y=m(X-4)\land m^2<\frac{1}{12} \land X=4 \right)\right.\\
&\lor \left.\left(X=\frac{16m^2}{4m^2+1} \land Y=m(X-4)\land m^2<\frac{1}{12} \land X \neq 4\right)\right]\\
\Longleftrightarrow~&\exists m\left[X=\frac{16m^2}{4m^2+1} \land m=\frac{Y}{X-4}\land m^2<\frac{1}{12} \land X \neq 4\right] \tag{0}\\
\Longleftrightarrow~&\exists m\left[m^2=\frac{1}{4}\left(\frac{X}{4-X}\right)\land m=\frac{Y}{X-4}\land m^2<\frac{1}{12} \land X \neq 4 \right] \tag{1}\\
\Longleftrightarrow~&\exists m \left[m^2=\frac{1}{4}\left(\frac{X}{4-X}\right) \land m=\frac{Y}{X-4}\land \frac{1}{4}\left(\frac{X}{4-X}\right)<\frac{1}{12} \land X \neq 4 \right]\tag{2}\\
\Longleftrightarrow~&\left(\frac{Y}{X-4}\right)^2=\frac{1}{4}\left(\frac{X}{4-X}\right) \land \frac{1}{4}\left(\frac{X}{4-X}\right)<\frac{1}{12} \land X \neq 4 \tag{3}\\
\Longleftrightarrow~&\frac{(X-2)^2}{4}+Y^2 = 1 \land (X < 1 \lor 4 < X) \land X \neq 4\\
\Longleftrightarrow~&\left(\frac{(X-2)^2}{4}+Y^2 = 1 \land X \neq 4 \land X < 1 \right) \lor \left( \frac{(X-2)^2}{4}+Y^2 = 1\land X \neq 4 \land 4 < X \right) \\
\Longleftrightarrow~&\frac{(X-2)^2}{4}+Y^2 = 1 \land X \neq 4 \land X < 1 \tag{\(\ast\)}\\
\Longleftrightarrow~&\frac{(X-2)^2}{4}+Y^2 = 1 \land 0 \leq X < 1 \tag{\(\ast\)}
\end{align*}

\((1)\)は第一式を\(m^2\)について解いた
\((2)\)代入法
\((3)\)\(\exists m\)の処理

\((\ast)\)は下図による.

 

◆軌跡と同値変形その1

\begin{align*}
&\exists m \left[X=\frac{16m^2}{4m^2+1} \land Y=m(X-4)\land m^2<\frac{1}{12}\right]\\
\Longleftrightarrow~&\exists m\left[X=\frac{16m^2}{4m^2+1} \land Y=m(X-4)\land m^2<\frac{1}{12} \land (X=4 \lor X \neq 4)\right]\\
\Longleftrightarrow~&\exists m\left[\left(X=\frac{16m^2}{4m^2+1} \land Y=m(X-4)\land m^2<\frac{1}{12} \land X=4 \right)\right.\\
&\lor \left.\left(X=\frac{16m^2}{4m^2+1} \land Y=m(X-4)\land m^2<\frac{1}{12} \land X \neq 4\right)\right]\\
\Longleftrightarrow~&\exists m\left[X=\frac{16m^2}{4m^2+1} \land m=\frac{Y}{X-4}\land m^2<\frac{1}{12} \land X \neq 4\right]\tag{0}\\
\Longleftrightarrow~&X=\frac{16\left(\frac{Y}{X-4}\right)^2}{4\left(\frac{Y}{X-4}\right)^2+1} \land \left(\frac{Y}{X-4}\right)^2<\frac{1}{12} \land X \neq 4\\
\Longleftrightarrow~&\frac{(X-2)^2}{4}+Y^2 = 1 \land \left|\frac{Y}{X-4}\right|<\frac{1}{\sqrt{12}} \land X \neq 4\\
\Longleftrightarrow~&\frac{(X-2)^2}{4}+Y^2 = 1 \land |Y|<\frac{1}{\sqrt{12}}|X-4| \land X \neq 4\\
\Longleftrightarrow~&\frac{(X-2)^2}{4}+Y^2 = 1 \land 0 \leq X < 1 \land X \neq 4 &\tag{\(\ast\)}\\
\Longleftrightarrow~&\frac{(X-2)^2}{4}+Y^2 = 1 \land 0 \leq X < 1\\
\end{align*}

\((\ast)\)は下図による.


\((0)\)以降の別変形はこちら

© 2024 佐々木数学塾, All rights reserved.